LSAT and Law School Admissions Forum

Get expert LSAT preparation and law school admissions advice from PowerScore Test Preparation.

User avatar
 Chandler H
PowerScore Staff
  • PowerScore Staff
  • Posts: 105
  • Joined: Feb 09, 2024
|
#106156
cd1010 wrote: Wed Apr 24, 2024 6:04 am Is it ok to say that another reason why B is wrong is because it is not comparative? I'm not sure if this point is generalizable, or particular to this question. The stimulus is presenting a paradoxical issue relating to two cities. In B, we're only presented with information regarding one city, but no additional info about how this new point relates to the other. Sure, "temperatures in Charlesville drop steeply at night", but we don't know by how much, and how much relative to the temp in Taychester. And as the explanation said, T is always colder anyway.
Hi cd1010,

Yes, you're right! The stimulus tells us that Taychester is always at least 10 degrees colder. Therefore, if temperatures drop steeply in Charlesville, then they're dropping steeply in Taychester as well.
 lsatstudent99966
  • Posts: 148
  • Joined: Jul 29, 2024
|
#110877
I have one question regarding (A).

It only tells us that “heat loss due to wind” is less in Taychester.

It didn't even tell us that “heat loss in general” is less in Taychester.

Isn’t it possible that heat loss, in general, is still the same (or even more) in Taychester, since other factors might also influence heat loss (for example, as brought up by (E), the insulation factor)?
User avatar
 Amber Thomas
PowerScore Staff
  • PowerScore Staff
  • Posts: 190
  • Joined: Oct 03, 2024
|
#111130
Hi Lsatstudent99966!

To resolve the apparent paradox, we simply need some sort of explanation that gives us a logical reason as to why winter heating expenses would be higher in Charlesville despite their higher temperatures. An explanation wouldn't even need to go as far as saying that heat loss in general is lower in Taychester than in Charlesville. If there is more heat loss due to wind in Charlesville, that gives us a sufficient, rational reason as to why their heating expenses are higher in spite of their higher temperatures. It feels colder due to wind chill, therefore, Charlesville spends more on winter heating than does Taychester.

I hope this helps!
 lsatstudent99966
  • Posts: 148
  • Joined: Jul 29, 2024
|
#111184
Amber Thomas wrote: Wed Dec 18, 2024 2:55 pm Hi Lsatstudent99966!

To resolve the apparent paradox, we simply need some sort of explanation that gives us a logical reason as to why winter heating expenses would be higher in Charlesville despite their higher temperatures. An explanation wouldn't even need to go as far as saying that heat loss in general is lower in Taychester than in Charlesville. If there is more heat loss due to wind in Charlesville, that gives us a sufficient, rational reason as to why their heating expenses are higher in spite of their higher temperatures. It feels colder due to wind chill, therefore, Charlesville spends more on winter heating than does Taychester.

I hope this helps!
Hi Thomas,

Thank you so much!

Am I right in saying that this is because in these paradox questions, we don't need an answer choice that definitively proves a solution to the paradox? We just need some reasons to reconcile the paradox.

So while it may be true that heat loss is generally lower in Taychester than in Charlesville, and if that is true, then the fact that there is more heat loss due to wind in Charlesville is not the answer to the puzzle. But it doesn't matter, because the possibility that heat loss is generally lower in Taychester than in Charlesville already exists (answer choice (A) adds nothing to that possibility), and just knowing from (A) that there is more heat loss due to wind in Charlesville brings us closer to solving the paradox?
User avatar
 Amber Thomas
PowerScore Staff
  • PowerScore Staff
  • Posts: 190
  • Joined: Oct 03, 2024
|
#111784
Hi LSATStudent!

You're exactly right, we don't need an answer that definitively solves the paradox-- we just need some potential reason that the paradox may not, in fact, be a paradox. Your explanation for this specific questions works well too!

I hope this helps!
 lsatstudent99966
  • Posts: 148
  • Joined: Jul 29, 2024
|
#111817
Amber Thomas wrote: Wed Feb 05, 2025 1:50 pm Hi LSATStudent!

You're exactly right, we don't need an answer that definitively solves the paradox-- we just need some potential reason that the paradox may not, in fact, be a paradox. Your explanation for this specific questions works well too!

I hope this helps!
Thank you so much Amber!

Get the most out of your LSAT Prep Plus subscription.

Analyze and track your performance with our Testing and Analytics Package.